Inequality of norms












0












$begingroup$


Is it obvious that in $mathbb R^n, forall p,q text{ s.t. } p leq q$



$$ vert vert xvert vert_{p} leq vert vert xvert vert_{q}, text{where }vert vert xvert vert_{q} = (sum_0^n x^q)^{frac 1 q }$$
If yes, how to prove it ? I was thinking about the classical inequalities like Minkowski, Holder or Cauchy Schwartz but none seems to work here.



Moreover, is this inequality still true for any norm ? Like the norm $L_p $ and $L_q$ ? (I mean by this $ (int vert f(x)^q vert dx ) ^{frac 1 q } $ )



In other words, is it true for a space of infinite dimension ?



What about the infinite norm ? is it also always bigger than any other one ? (this one I know is false for the case $mathbb R^n $, but it is true for functions spaces like the space of continuous functions...)





P.S. I'm sorry, I have not yet studied functionnal analysis. I'm asking this for my metric space class where we are proving that some spaces are complete, and I'm wondering when is it possible to use the very usefull inequality of norm I was talking about.



I'm asking this because intuitively, it seems right :
enter image description here










share|cite|improve this question









$endgroup$

















    0












    $begingroup$


    Is it obvious that in $mathbb R^n, forall p,q text{ s.t. } p leq q$



    $$ vert vert xvert vert_{p} leq vert vert xvert vert_{q}, text{where }vert vert xvert vert_{q} = (sum_0^n x^q)^{frac 1 q }$$
    If yes, how to prove it ? I was thinking about the classical inequalities like Minkowski, Holder or Cauchy Schwartz but none seems to work here.



    Moreover, is this inequality still true for any norm ? Like the norm $L_p $ and $L_q$ ? (I mean by this $ (int vert f(x)^q vert dx ) ^{frac 1 q } $ )



    In other words, is it true for a space of infinite dimension ?



    What about the infinite norm ? is it also always bigger than any other one ? (this one I know is false for the case $mathbb R^n $, but it is true for functions spaces like the space of continuous functions...)





    P.S. I'm sorry, I have not yet studied functionnal analysis. I'm asking this for my metric space class where we are proving that some spaces are complete, and I'm wondering when is it possible to use the very usefull inequality of norm I was talking about.



    I'm asking this because intuitively, it seems right :
    enter image description here










    share|cite|improve this question









    $endgroup$















      0












      0








      0





      $begingroup$


      Is it obvious that in $mathbb R^n, forall p,q text{ s.t. } p leq q$



      $$ vert vert xvert vert_{p} leq vert vert xvert vert_{q}, text{where }vert vert xvert vert_{q} = (sum_0^n x^q)^{frac 1 q }$$
      If yes, how to prove it ? I was thinking about the classical inequalities like Minkowski, Holder or Cauchy Schwartz but none seems to work here.



      Moreover, is this inequality still true for any norm ? Like the norm $L_p $ and $L_q$ ? (I mean by this $ (int vert f(x)^q vert dx ) ^{frac 1 q } $ )



      In other words, is it true for a space of infinite dimension ?



      What about the infinite norm ? is it also always bigger than any other one ? (this one I know is false for the case $mathbb R^n $, but it is true for functions spaces like the space of continuous functions...)





      P.S. I'm sorry, I have not yet studied functionnal analysis. I'm asking this for my metric space class where we are proving that some spaces are complete, and I'm wondering when is it possible to use the very usefull inequality of norm I was talking about.



      I'm asking this because intuitively, it seems right :
      enter image description here










      share|cite|improve this question









      $endgroup$




      Is it obvious that in $mathbb R^n, forall p,q text{ s.t. } p leq q$



      $$ vert vert xvert vert_{p} leq vert vert xvert vert_{q}, text{where }vert vert xvert vert_{q} = (sum_0^n x^q)^{frac 1 q }$$
      If yes, how to prove it ? I was thinking about the classical inequalities like Minkowski, Holder or Cauchy Schwartz but none seems to work here.



      Moreover, is this inequality still true for any norm ? Like the norm $L_p $ and $L_q$ ? (I mean by this $ (int vert f(x)^q vert dx ) ^{frac 1 q } $ )



      In other words, is it true for a space of infinite dimension ?



      What about the infinite norm ? is it also always bigger than any other one ? (this one I know is false for the case $mathbb R^n $, but it is true for functions spaces like the space of continuous functions...)





      P.S. I'm sorry, I have not yet studied functionnal analysis. I'm asking this for my metric space class where we are proving that some spaces are complete, and I'm wondering when is it possible to use the very usefull inequality of norm I was talking about.



      I'm asking this because intuitively, it seems right :
      enter image description here







      real-analysis norm normed-spaces






      share|cite|improve this question













      share|cite|improve this question











      share|cite|improve this question




      share|cite|improve this question










      asked Jan 23 at 18:37









      Marine GalantinMarine Galantin

      875319




      875319






















          1 Answer
          1






          active

          oldest

          votes


















          1












          $begingroup$

          Actually, direction should be reversed: it holds that
          $$
          |x|_pge |x|_q,quad 1le p< qleinfty.tag{*}
          $$
          What your figure shows us is that
          $$
          B_psubset B_q,quad p< qtag{**}
          $$
          where $B_r$ is a unit ball in $r$-norm. It is easy to see that $text{(*)}$ is equivalent to $text{(**)}$.



          Now, we prove $text{(*)}$. By the homogeneity of norms, it is sufficient to show that
          $$
          |x|_qle 1
          $$
          provided that $|x|_p= 1$. By the assumption, we have
          $$
          |x_i|^ple |x_1|^p+|x_2|^p+cdots +|x_n|^p=1
          $$
          for all $ile n$. Hence $|x_i|le 1$ and it follows
          $$
          |x_i|^qle |x_i|^p,quad i=1,2,ldots,n
          $$
          for $q<infty$, and $|x|_infty le 1$ for $q=infty$.
          Summing over $i$ yields
          $$
          |x|_q^q =|x_1|^q+|x_2|^q+cdots +|x_n|^qle |x_1|^p+|x_2|^p+cdots +|x_n|^p=1
          $$
          for $q<infty$. This proves $|x|_qle 1$ for all $1le p< qle infty$ as desired.



          Note: The inequality can be easily extended to $Bbb R^{Bbb N}$ equipped with $l^p$-norms, i.e. $l^p(Bbb N)$. And reverse inequality is true if we are on a finite measure space, i.e.
          $$
          |f|_ple mu(Omega)^{1/p-1/q}|f|_q,quad 1le p<qleinfty.
          $$
          holds for $fin L^q(Omega)subseteq L^p(Omega)$. (It a direct consequence of Jensen's inequality.) But in a general measure space, in particular with infinite total measure such as $L^p(Bbb R)$, none of the relationship $$
          |f|_ple C|f|_q
          $$
          nor
          $$
          |f|_qle C|f|_p
          $$
          is true for any $C>0$. (So there is no inclusion between $L^p(Bbb R)$-spaces. I'll omit further explanation, but one maybe able to find examples of this.) Thus the fact that $$l^p(Bbb N)subseteq l^q(Bbb N), quad 1le p<qle infty$$ is a special property, which can be attributed to certain properties of counting measure.






          share|cite|improve this answer











          $endgroup$













          • $begingroup$
            Are you sure? In my lecture it s written : $$ abs(f_m(t) - f_n(t)) leq vert vert f_m- f_n vert vert_{infty} $$ and also for any element in the space of sequences $l^2_{ mathbb F} $ : $$ abs( u_j^{(n) } - u_j^{(m) }) leq vert vert u^{(n) } - u^{(m) } vert vert_{2} $$
            $endgroup$
            – Marine Galantin
            Jan 23 at 19:20












          • $begingroup$
            And what about the other points I was talking about? Maybe it s linked to my other comment. Please be as precise as you can, i m baffled with all of this
            $endgroup$
            – Marine Galantin
            Jan 23 at 19:24






          • 1




            $begingroup$
            @MarineGalantin Yes, I'm so sure. If you are suspicious, why don't you calculate actual $p$-norms of a vector, say $(1,3,4)$? I think I fully explained why $|x|_p ge |x|_q$ is true. And about those two facts, yes, they are true $textbf{independently}$ of the inequality I showed. Those are just trivial pointwise bound obtained from the definition of sup-norm and $2$-norm. And they have nothing to do with the inequality discussed.
            $endgroup$
            – Song
            Jan 23 at 19:43








          • 1




            $begingroup$
            @MarineGalantin The direction of the inequality is clear if you remember the obvious endpoint $$ |x|_{ell^infty} le |x|_{ell^1}$$ that cannot be reversed
            $endgroup$
            – Calvin Khor
            Jan 23 at 20:36






          • 1




            $begingroup$
            @MarineGalantin Yes, surely! To add some more explanation (using symbolic manipulations), let us denote $ncdot B_p ={ncdot xin l^p(Bbb N);:;x in B_p}$ (it is just a ball of radius $n$.) Then,$$ forall nge 1,quad ncdot B_psubset ncdot B_qsubset l^q(Bbb N).$$ Since $bigcup_{nge 1} ncdot B_p = l^p(Bbb N)$ , it follows $$l^p(Bbb N)subset l^q(Bbb N).$$
            $endgroup$
            – Song
            Jan 23 at 21:02













          Your Answer





          StackExchange.ifUsing("editor", function () {
          return StackExchange.using("mathjaxEditing", function () {
          StackExchange.MarkdownEditor.creationCallbacks.add(function (editor, postfix) {
          StackExchange.mathjaxEditing.prepareWmdForMathJax(editor, postfix, [["$", "$"], ["\\(","\\)"]]);
          });
          });
          }, "mathjax-editing");

          StackExchange.ready(function() {
          var channelOptions = {
          tags: "".split(" "),
          id: "69"
          };
          initTagRenderer("".split(" "), "".split(" "), channelOptions);

          StackExchange.using("externalEditor", function() {
          // Have to fire editor after snippets, if snippets enabled
          if (StackExchange.settings.snippets.snippetsEnabled) {
          StackExchange.using("snippets", function() {
          createEditor();
          });
          }
          else {
          createEditor();
          }
          });

          function createEditor() {
          StackExchange.prepareEditor({
          heartbeatType: 'answer',
          autoActivateHeartbeat: false,
          convertImagesToLinks: true,
          noModals: true,
          showLowRepImageUploadWarning: true,
          reputationToPostImages: 10,
          bindNavPrevention: true,
          postfix: "",
          imageUploader: {
          brandingHtml: "Powered by u003ca class="icon-imgur-white" href="https://imgur.com/"u003eu003c/au003e",
          contentPolicyHtml: "User contributions licensed under u003ca href="https://creativecommons.org/licenses/by-sa/3.0/"u003ecc by-sa 3.0 with attribution requiredu003c/au003e u003ca href="https://stackoverflow.com/legal/content-policy"u003e(content policy)u003c/au003e",
          allowUrls: true
          },
          noCode: true, onDemand: true,
          discardSelector: ".discard-answer"
          ,immediatelyShowMarkdownHelp:true
          });


          }
          });














          draft saved

          draft discarded


















          StackExchange.ready(
          function () {
          StackExchange.openid.initPostLogin('.new-post-login', 'https%3a%2f%2fmath.stackexchange.com%2fquestions%2f3084881%2finequality-of-norms%23new-answer', 'question_page');
          }
          );

          Post as a guest















          Required, but never shown

























          1 Answer
          1






          active

          oldest

          votes








          1 Answer
          1






          active

          oldest

          votes









          active

          oldest

          votes






          active

          oldest

          votes









          1












          $begingroup$

          Actually, direction should be reversed: it holds that
          $$
          |x|_pge |x|_q,quad 1le p< qleinfty.tag{*}
          $$
          What your figure shows us is that
          $$
          B_psubset B_q,quad p< qtag{**}
          $$
          where $B_r$ is a unit ball in $r$-norm. It is easy to see that $text{(*)}$ is equivalent to $text{(**)}$.



          Now, we prove $text{(*)}$. By the homogeneity of norms, it is sufficient to show that
          $$
          |x|_qle 1
          $$
          provided that $|x|_p= 1$. By the assumption, we have
          $$
          |x_i|^ple |x_1|^p+|x_2|^p+cdots +|x_n|^p=1
          $$
          for all $ile n$. Hence $|x_i|le 1$ and it follows
          $$
          |x_i|^qle |x_i|^p,quad i=1,2,ldots,n
          $$
          for $q<infty$, and $|x|_infty le 1$ for $q=infty$.
          Summing over $i$ yields
          $$
          |x|_q^q =|x_1|^q+|x_2|^q+cdots +|x_n|^qle |x_1|^p+|x_2|^p+cdots +|x_n|^p=1
          $$
          for $q<infty$. This proves $|x|_qle 1$ for all $1le p< qle infty$ as desired.



          Note: The inequality can be easily extended to $Bbb R^{Bbb N}$ equipped with $l^p$-norms, i.e. $l^p(Bbb N)$. And reverse inequality is true if we are on a finite measure space, i.e.
          $$
          |f|_ple mu(Omega)^{1/p-1/q}|f|_q,quad 1le p<qleinfty.
          $$
          holds for $fin L^q(Omega)subseteq L^p(Omega)$. (It a direct consequence of Jensen's inequality.) But in a general measure space, in particular with infinite total measure such as $L^p(Bbb R)$, none of the relationship $$
          |f|_ple C|f|_q
          $$
          nor
          $$
          |f|_qle C|f|_p
          $$
          is true for any $C>0$. (So there is no inclusion between $L^p(Bbb R)$-spaces. I'll omit further explanation, but one maybe able to find examples of this.) Thus the fact that $$l^p(Bbb N)subseteq l^q(Bbb N), quad 1le p<qle infty$$ is a special property, which can be attributed to certain properties of counting measure.






          share|cite|improve this answer











          $endgroup$













          • $begingroup$
            Are you sure? In my lecture it s written : $$ abs(f_m(t) - f_n(t)) leq vert vert f_m- f_n vert vert_{infty} $$ and also for any element in the space of sequences $l^2_{ mathbb F} $ : $$ abs( u_j^{(n) } - u_j^{(m) }) leq vert vert u^{(n) } - u^{(m) } vert vert_{2} $$
            $endgroup$
            – Marine Galantin
            Jan 23 at 19:20












          • $begingroup$
            And what about the other points I was talking about? Maybe it s linked to my other comment. Please be as precise as you can, i m baffled with all of this
            $endgroup$
            – Marine Galantin
            Jan 23 at 19:24






          • 1




            $begingroup$
            @MarineGalantin Yes, I'm so sure. If you are suspicious, why don't you calculate actual $p$-norms of a vector, say $(1,3,4)$? I think I fully explained why $|x|_p ge |x|_q$ is true. And about those two facts, yes, they are true $textbf{independently}$ of the inequality I showed. Those are just trivial pointwise bound obtained from the definition of sup-norm and $2$-norm. And they have nothing to do with the inequality discussed.
            $endgroup$
            – Song
            Jan 23 at 19:43








          • 1




            $begingroup$
            @MarineGalantin The direction of the inequality is clear if you remember the obvious endpoint $$ |x|_{ell^infty} le |x|_{ell^1}$$ that cannot be reversed
            $endgroup$
            – Calvin Khor
            Jan 23 at 20:36






          • 1




            $begingroup$
            @MarineGalantin Yes, surely! To add some more explanation (using symbolic manipulations), let us denote $ncdot B_p ={ncdot xin l^p(Bbb N);:;x in B_p}$ (it is just a ball of radius $n$.) Then,$$ forall nge 1,quad ncdot B_psubset ncdot B_qsubset l^q(Bbb N).$$ Since $bigcup_{nge 1} ncdot B_p = l^p(Bbb N)$ , it follows $$l^p(Bbb N)subset l^q(Bbb N).$$
            $endgroup$
            – Song
            Jan 23 at 21:02


















          1












          $begingroup$

          Actually, direction should be reversed: it holds that
          $$
          |x|_pge |x|_q,quad 1le p< qleinfty.tag{*}
          $$
          What your figure shows us is that
          $$
          B_psubset B_q,quad p< qtag{**}
          $$
          where $B_r$ is a unit ball in $r$-norm. It is easy to see that $text{(*)}$ is equivalent to $text{(**)}$.



          Now, we prove $text{(*)}$. By the homogeneity of norms, it is sufficient to show that
          $$
          |x|_qle 1
          $$
          provided that $|x|_p= 1$. By the assumption, we have
          $$
          |x_i|^ple |x_1|^p+|x_2|^p+cdots +|x_n|^p=1
          $$
          for all $ile n$. Hence $|x_i|le 1$ and it follows
          $$
          |x_i|^qle |x_i|^p,quad i=1,2,ldots,n
          $$
          for $q<infty$, and $|x|_infty le 1$ for $q=infty$.
          Summing over $i$ yields
          $$
          |x|_q^q =|x_1|^q+|x_2|^q+cdots +|x_n|^qle |x_1|^p+|x_2|^p+cdots +|x_n|^p=1
          $$
          for $q<infty$. This proves $|x|_qle 1$ for all $1le p< qle infty$ as desired.



          Note: The inequality can be easily extended to $Bbb R^{Bbb N}$ equipped with $l^p$-norms, i.e. $l^p(Bbb N)$. And reverse inequality is true if we are on a finite measure space, i.e.
          $$
          |f|_ple mu(Omega)^{1/p-1/q}|f|_q,quad 1le p<qleinfty.
          $$
          holds for $fin L^q(Omega)subseteq L^p(Omega)$. (It a direct consequence of Jensen's inequality.) But in a general measure space, in particular with infinite total measure such as $L^p(Bbb R)$, none of the relationship $$
          |f|_ple C|f|_q
          $$
          nor
          $$
          |f|_qle C|f|_p
          $$
          is true for any $C>0$. (So there is no inclusion between $L^p(Bbb R)$-spaces. I'll omit further explanation, but one maybe able to find examples of this.) Thus the fact that $$l^p(Bbb N)subseteq l^q(Bbb N), quad 1le p<qle infty$$ is a special property, which can be attributed to certain properties of counting measure.






          share|cite|improve this answer











          $endgroup$













          • $begingroup$
            Are you sure? In my lecture it s written : $$ abs(f_m(t) - f_n(t)) leq vert vert f_m- f_n vert vert_{infty} $$ and also for any element in the space of sequences $l^2_{ mathbb F} $ : $$ abs( u_j^{(n) } - u_j^{(m) }) leq vert vert u^{(n) } - u^{(m) } vert vert_{2} $$
            $endgroup$
            – Marine Galantin
            Jan 23 at 19:20












          • $begingroup$
            And what about the other points I was talking about? Maybe it s linked to my other comment. Please be as precise as you can, i m baffled with all of this
            $endgroup$
            – Marine Galantin
            Jan 23 at 19:24






          • 1




            $begingroup$
            @MarineGalantin Yes, I'm so sure. If you are suspicious, why don't you calculate actual $p$-norms of a vector, say $(1,3,4)$? I think I fully explained why $|x|_p ge |x|_q$ is true. And about those two facts, yes, they are true $textbf{independently}$ of the inequality I showed. Those are just trivial pointwise bound obtained from the definition of sup-norm and $2$-norm. And they have nothing to do with the inequality discussed.
            $endgroup$
            – Song
            Jan 23 at 19:43








          • 1




            $begingroup$
            @MarineGalantin The direction of the inequality is clear if you remember the obvious endpoint $$ |x|_{ell^infty} le |x|_{ell^1}$$ that cannot be reversed
            $endgroup$
            – Calvin Khor
            Jan 23 at 20:36






          • 1




            $begingroup$
            @MarineGalantin Yes, surely! To add some more explanation (using symbolic manipulations), let us denote $ncdot B_p ={ncdot xin l^p(Bbb N);:;x in B_p}$ (it is just a ball of radius $n$.) Then,$$ forall nge 1,quad ncdot B_psubset ncdot B_qsubset l^q(Bbb N).$$ Since $bigcup_{nge 1} ncdot B_p = l^p(Bbb N)$ , it follows $$l^p(Bbb N)subset l^q(Bbb N).$$
            $endgroup$
            – Song
            Jan 23 at 21:02
















          1












          1








          1





          $begingroup$

          Actually, direction should be reversed: it holds that
          $$
          |x|_pge |x|_q,quad 1le p< qleinfty.tag{*}
          $$
          What your figure shows us is that
          $$
          B_psubset B_q,quad p< qtag{**}
          $$
          where $B_r$ is a unit ball in $r$-norm. It is easy to see that $text{(*)}$ is equivalent to $text{(**)}$.



          Now, we prove $text{(*)}$. By the homogeneity of norms, it is sufficient to show that
          $$
          |x|_qle 1
          $$
          provided that $|x|_p= 1$. By the assumption, we have
          $$
          |x_i|^ple |x_1|^p+|x_2|^p+cdots +|x_n|^p=1
          $$
          for all $ile n$. Hence $|x_i|le 1$ and it follows
          $$
          |x_i|^qle |x_i|^p,quad i=1,2,ldots,n
          $$
          for $q<infty$, and $|x|_infty le 1$ for $q=infty$.
          Summing over $i$ yields
          $$
          |x|_q^q =|x_1|^q+|x_2|^q+cdots +|x_n|^qle |x_1|^p+|x_2|^p+cdots +|x_n|^p=1
          $$
          for $q<infty$. This proves $|x|_qle 1$ for all $1le p< qle infty$ as desired.



          Note: The inequality can be easily extended to $Bbb R^{Bbb N}$ equipped with $l^p$-norms, i.e. $l^p(Bbb N)$. And reverse inequality is true if we are on a finite measure space, i.e.
          $$
          |f|_ple mu(Omega)^{1/p-1/q}|f|_q,quad 1le p<qleinfty.
          $$
          holds for $fin L^q(Omega)subseteq L^p(Omega)$. (It a direct consequence of Jensen's inequality.) But in a general measure space, in particular with infinite total measure such as $L^p(Bbb R)$, none of the relationship $$
          |f|_ple C|f|_q
          $$
          nor
          $$
          |f|_qle C|f|_p
          $$
          is true for any $C>0$. (So there is no inclusion between $L^p(Bbb R)$-spaces. I'll omit further explanation, but one maybe able to find examples of this.) Thus the fact that $$l^p(Bbb N)subseteq l^q(Bbb N), quad 1le p<qle infty$$ is a special property, which can be attributed to certain properties of counting measure.






          share|cite|improve this answer











          $endgroup$



          Actually, direction should be reversed: it holds that
          $$
          |x|_pge |x|_q,quad 1le p< qleinfty.tag{*}
          $$
          What your figure shows us is that
          $$
          B_psubset B_q,quad p< qtag{**}
          $$
          where $B_r$ is a unit ball in $r$-norm. It is easy to see that $text{(*)}$ is equivalent to $text{(**)}$.



          Now, we prove $text{(*)}$. By the homogeneity of norms, it is sufficient to show that
          $$
          |x|_qle 1
          $$
          provided that $|x|_p= 1$. By the assumption, we have
          $$
          |x_i|^ple |x_1|^p+|x_2|^p+cdots +|x_n|^p=1
          $$
          for all $ile n$. Hence $|x_i|le 1$ and it follows
          $$
          |x_i|^qle |x_i|^p,quad i=1,2,ldots,n
          $$
          for $q<infty$, and $|x|_infty le 1$ for $q=infty$.
          Summing over $i$ yields
          $$
          |x|_q^q =|x_1|^q+|x_2|^q+cdots +|x_n|^qle |x_1|^p+|x_2|^p+cdots +|x_n|^p=1
          $$
          for $q<infty$. This proves $|x|_qle 1$ for all $1le p< qle infty$ as desired.



          Note: The inequality can be easily extended to $Bbb R^{Bbb N}$ equipped with $l^p$-norms, i.e. $l^p(Bbb N)$. And reverse inequality is true if we are on a finite measure space, i.e.
          $$
          |f|_ple mu(Omega)^{1/p-1/q}|f|_q,quad 1le p<qleinfty.
          $$
          holds for $fin L^q(Omega)subseteq L^p(Omega)$. (It a direct consequence of Jensen's inequality.) But in a general measure space, in particular with infinite total measure such as $L^p(Bbb R)$, none of the relationship $$
          |f|_ple C|f|_q
          $$
          nor
          $$
          |f|_qle C|f|_p
          $$
          is true for any $C>0$. (So there is no inclusion between $L^p(Bbb R)$-spaces. I'll omit further explanation, but one maybe able to find examples of this.) Thus the fact that $$l^p(Bbb N)subseteq l^q(Bbb N), quad 1le p<qle infty$$ is a special property, which can be attributed to certain properties of counting measure.







          share|cite|improve this answer














          share|cite|improve this answer



          share|cite|improve this answer








          edited Jan 23 at 19:56

























          answered Jan 23 at 18:52









          SongSong

          17.5k21246




          17.5k21246












          • $begingroup$
            Are you sure? In my lecture it s written : $$ abs(f_m(t) - f_n(t)) leq vert vert f_m- f_n vert vert_{infty} $$ and also for any element in the space of sequences $l^2_{ mathbb F} $ : $$ abs( u_j^{(n) } - u_j^{(m) }) leq vert vert u^{(n) } - u^{(m) } vert vert_{2} $$
            $endgroup$
            – Marine Galantin
            Jan 23 at 19:20












          • $begingroup$
            And what about the other points I was talking about? Maybe it s linked to my other comment. Please be as precise as you can, i m baffled with all of this
            $endgroup$
            – Marine Galantin
            Jan 23 at 19:24






          • 1




            $begingroup$
            @MarineGalantin Yes, I'm so sure. If you are suspicious, why don't you calculate actual $p$-norms of a vector, say $(1,3,4)$? I think I fully explained why $|x|_p ge |x|_q$ is true. And about those two facts, yes, they are true $textbf{independently}$ of the inequality I showed. Those are just trivial pointwise bound obtained from the definition of sup-norm and $2$-norm. And they have nothing to do with the inequality discussed.
            $endgroup$
            – Song
            Jan 23 at 19:43








          • 1




            $begingroup$
            @MarineGalantin The direction of the inequality is clear if you remember the obvious endpoint $$ |x|_{ell^infty} le |x|_{ell^1}$$ that cannot be reversed
            $endgroup$
            – Calvin Khor
            Jan 23 at 20:36






          • 1




            $begingroup$
            @MarineGalantin Yes, surely! To add some more explanation (using symbolic manipulations), let us denote $ncdot B_p ={ncdot xin l^p(Bbb N);:;x in B_p}$ (it is just a ball of radius $n$.) Then,$$ forall nge 1,quad ncdot B_psubset ncdot B_qsubset l^q(Bbb N).$$ Since $bigcup_{nge 1} ncdot B_p = l^p(Bbb N)$ , it follows $$l^p(Bbb N)subset l^q(Bbb N).$$
            $endgroup$
            – Song
            Jan 23 at 21:02




















          • $begingroup$
            Are you sure? In my lecture it s written : $$ abs(f_m(t) - f_n(t)) leq vert vert f_m- f_n vert vert_{infty} $$ and also for any element in the space of sequences $l^2_{ mathbb F} $ : $$ abs( u_j^{(n) } - u_j^{(m) }) leq vert vert u^{(n) } - u^{(m) } vert vert_{2} $$
            $endgroup$
            – Marine Galantin
            Jan 23 at 19:20












          • $begingroup$
            And what about the other points I was talking about? Maybe it s linked to my other comment. Please be as precise as you can, i m baffled with all of this
            $endgroup$
            – Marine Galantin
            Jan 23 at 19:24






          • 1




            $begingroup$
            @MarineGalantin Yes, I'm so sure. If you are suspicious, why don't you calculate actual $p$-norms of a vector, say $(1,3,4)$? I think I fully explained why $|x|_p ge |x|_q$ is true. And about those two facts, yes, they are true $textbf{independently}$ of the inequality I showed. Those are just trivial pointwise bound obtained from the definition of sup-norm and $2$-norm. And they have nothing to do with the inequality discussed.
            $endgroup$
            – Song
            Jan 23 at 19:43








          • 1




            $begingroup$
            @MarineGalantin The direction of the inequality is clear if you remember the obvious endpoint $$ |x|_{ell^infty} le |x|_{ell^1}$$ that cannot be reversed
            $endgroup$
            – Calvin Khor
            Jan 23 at 20:36






          • 1




            $begingroup$
            @MarineGalantin Yes, surely! To add some more explanation (using symbolic manipulations), let us denote $ncdot B_p ={ncdot xin l^p(Bbb N);:;x in B_p}$ (it is just a ball of radius $n$.) Then,$$ forall nge 1,quad ncdot B_psubset ncdot B_qsubset l^q(Bbb N).$$ Since $bigcup_{nge 1} ncdot B_p = l^p(Bbb N)$ , it follows $$l^p(Bbb N)subset l^q(Bbb N).$$
            $endgroup$
            – Song
            Jan 23 at 21:02


















          $begingroup$
          Are you sure? In my lecture it s written : $$ abs(f_m(t) - f_n(t)) leq vert vert f_m- f_n vert vert_{infty} $$ and also for any element in the space of sequences $l^2_{ mathbb F} $ : $$ abs( u_j^{(n) } - u_j^{(m) }) leq vert vert u^{(n) } - u^{(m) } vert vert_{2} $$
          $endgroup$
          – Marine Galantin
          Jan 23 at 19:20






          $begingroup$
          Are you sure? In my lecture it s written : $$ abs(f_m(t) - f_n(t)) leq vert vert f_m- f_n vert vert_{infty} $$ and also for any element in the space of sequences $l^2_{ mathbb F} $ : $$ abs( u_j^{(n) } - u_j^{(m) }) leq vert vert u^{(n) } - u^{(m) } vert vert_{2} $$
          $endgroup$
          – Marine Galantin
          Jan 23 at 19:20














          $begingroup$
          And what about the other points I was talking about? Maybe it s linked to my other comment. Please be as precise as you can, i m baffled with all of this
          $endgroup$
          – Marine Galantin
          Jan 23 at 19:24




          $begingroup$
          And what about the other points I was talking about? Maybe it s linked to my other comment. Please be as precise as you can, i m baffled with all of this
          $endgroup$
          – Marine Galantin
          Jan 23 at 19:24




          1




          1




          $begingroup$
          @MarineGalantin Yes, I'm so sure. If you are suspicious, why don't you calculate actual $p$-norms of a vector, say $(1,3,4)$? I think I fully explained why $|x|_p ge |x|_q$ is true. And about those two facts, yes, they are true $textbf{independently}$ of the inequality I showed. Those are just trivial pointwise bound obtained from the definition of sup-norm and $2$-norm. And they have nothing to do with the inequality discussed.
          $endgroup$
          – Song
          Jan 23 at 19:43






          $begingroup$
          @MarineGalantin Yes, I'm so sure. If you are suspicious, why don't you calculate actual $p$-norms of a vector, say $(1,3,4)$? I think I fully explained why $|x|_p ge |x|_q$ is true. And about those two facts, yes, they are true $textbf{independently}$ of the inequality I showed. Those are just trivial pointwise bound obtained from the definition of sup-norm and $2$-norm. And they have nothing to do with the inequality discussed.
          $endgroup$
          – Song
          Jan 23 at 19:43






          1




          1




          $begingroup$
          @MarineGalantin The direction of the inequality is clear if you remember the obvious endpoint $$ |x|_{ell^infty} le |x|_{ell^1}$$ that cannot be reversed
          $endgroup$
          – Calvin Khor
          Jan 23 at 20:36




          $begingroup$
          @MarineGalantin The direction of the inequality is clear if you remember the obvious endpoint $$ |x|_{ell^infty} le |x|_{ell^1}$$ that cannot be reversed
          $endgroup$
          – Calvin Khor
          Jan 23 at 20:36




          1




          1




          $begingroup$
          @MarineGalantin Yes, surely! To add some more explanation (using symbolic manipulations), let us denote $ncdot B_p ={ncdot xin l^p(Bbb N);:;x in B_p}$ (it is just a ball of radius $n$.) Then,$$ forall nge 1,quad ncdot B_psubset ncdot B_qsubset l^q(Bbb N).$$ Since $bigcup_{nge 1} ncdot B_p = l^p(Bbb N)$ , it follows $$l^p(Bbb N)subset l^q(Bbb N).$$
          $endgroup$
          – Song
          Jan 23 at 21:02






          $begingroup$
          @MarineGalantin Yes, surely! To add some more explanation (using symbolic manipulations), let us denote $ncdot B_p ={ncdot xin l^p(Bbb N);:;x in B_p}$ (it is just a ball of radius $n$.) Then,$$ forall nge 1,quad ncdot B_psubset ncdot B_qsubset l^q(Bbb N).$$ Since $bigcup_{nge 1} ncdot B_p = l^p(Bbb N)$ , it follows $$l^p(Bbb N)subset l^q(Bbb N).$$
          $endgroup$
          – Song
          Jan 23 at 21:02




















          draft saved

          draft discarded




















































          Thanks for contributing an answer to Mathematics Stack Exchange!


          • Please be sure to answer the question. Provide details and share your research!

          But avoid



          • Asking for help, clarification, or responding to other answers.

          • Making statements based on opinion; back them up with references or personal experience.


          Use MathJax to format equations. MathJax reference.


          To learn more, see our tips on writing great answers.




          draft saved


          draft discarded














          StackExchange.ready(
          function () {
          StackExchange.openid.initPostLogin('.new-post-login', 'https%3a%2f%2fmath.stackexchange.com%2fquestions%2f3084881%2finequality-of-norms%23new-answer', 'question_page');
          }
          );

          Post as a guest















          Required, but never shown





















































          Required, but never shown














          Required, but never shown












          Required, but never shown







          Required, but never shown

































          Required, but never shown














          Required, but never shown












          Required, but never shown







          Required, but never shown







          Popular posts from this blog

          Can a sorcerer learn a 5th-level spell early by creating spell slots using the Font of Magic feature?

          Does disintegrating a polymorphed enemy still kill it after the 2018 errata?

          A Topological Invariant for $pi_3(U(n))$